Video není dostupné.
Omlouváme se.

The HARDEST Problem On India's Hardest Exam (JEE)

Sdílet
Vložit
  • čas přidán 16. 08. 2024

Komentáře • 166

  • @OscgrMaths
    @OscgrMaths  Před měsícem

    Hey everyone! Thanks for watching. I'm aware that at 1:38 my explanation was rushed and not very rigorous - sometimes when videos are too long I try and skip over important things like that! A more convincing argument may be to find the difference between the terms n+1 and n instead.

  • @Bangaudaala
    @Bangaudaala Před měsícem +18

    This is hard until you realize you only have a 50% chance to get it wrong

    • @aniketroy1745
      @aniketroy1745 Před měsícem +6

      There are 4 options tho, and there is -2 marks if you get the answers wrong

  • @yesa_absolutelt
    @yesa_absolutelt Před měsícem +6

    Try INMO problems. You will get them on aops, it is the toughest high school Math exam in India.

  • @ArtemKreimer
    @ArtemKreimer Před měsícem +1

    I'm not clear why you need to go through the process of solving for S_infinity.
    Consider π/(3sqrt3) > 3/(3sqrt3) = 1/sqrt3 > 1/3
    Consider S_1 = 1/(1^2+1x1+1^2) = 1/3
    Since S_1 = 1/3, we know that not all S_n can be > π/(3sqrt3) > 1/3
    Therefore, A

  • @risingredstone5949
    @risingredstone5949 Před měsícem +13

    Since the condition has to hold for all n's and either A or B has to be true. So all you have to do is substitute n = 1, you get S1 = 1/3. now, we can prove that pi/(3sqrt(3)) > 1/3 (pretty easy to do this). And hence the correct option is A.
    This question is to be done in under 1 minute in the exam. You cannot waste that much time on it.

  • @manijha1
    @manijha1 Před měsícem +1

    If those are the only two options, we can just take the value of S1 which is 1/3, this is already less than pi/(3*root(3)), so it's clear the second option can't be true. But of course, in the real exam there are four options, so it will be much harder. The solution is very elegant but one never has the time to construct such a solution in a exam like JEE :P

  • @DihinAmarasigha-up5hf
    @DihinAmarasigha-up5hf Před měsícem +40

    Loved the part where you turned the series into an integral using the basic integral definition....but I think the part where you told the function S (n) was increasing with n was not intuitive...because although the number of terms increase the individual terms are become smaller ...anyway that was my only issue otherwise it's pretty awesome video...

    • @OscgrMaths
      @OscgrMaths  Před měsícem +2

      Thanks so glad you enjoyed! Yeah I probably over explained that bit - just meant n approaching infinity would provide the maximum we're looking for. Thanks for the comment!

    • @andrewbuchanan5342
      @andrewbuchanan5342 Před měsícem +4

      @@OscgrMaths Yes not just a question of adding a summand s(n+1,n+1) when going from S_n to S_(n+1) also to compare s(k,n) with s(k,n+1) for all k =< n. Given that n is in the numerator & denominator for the summand s(k,n) I think there is a step missing to show that S_infy is indeed the maximum.

    • @silver6054
      @silver6054 Před měsícem +1

      @@andrewbuchanan5342 I thought the explanation was correct. As all terms are clearly positive, however small, S_m > S_n whenever m>n, simply because you are adding more positive terms when going from n to m, and so the max is indeed S_infy

    • @_Heb_
      @_Heb_ Před měsícem +1

      ⁠@@silver6054It's true that you have more positive terms in the S_m case than the S_n case when m > n, but the terms themselves are smaller due to the n^2 component in the denominator. You'd have to confirm that the terms being smaller isn't enough to offset there being more of them

    • @silver6054
      @silver6054 Před měsícem

      @@_Heb_ yes, you are right, totally missed the key point that S_m and S_n contain different terms, (i.e. the terms of S_n are not a subset of the terms of S_m). So yes, definitely need an argument that S_infy is a maximum

  • @user-xt8sh3vf5h
    @user-xt8sh3vf5h Před měsícem +1

    Love u bro from India

  • @advait8142
    @advait8142 Před měsícem +29

    beautiful use of the riemann sum! some of my friends are preparing for JEE in thr next few years and they have practice problems like this all the time. its a really challenging exam esp when you realise you can only dedicate 3ish minutes per question. anyways superb solution!

    • @OscgrMaths
      @OscgrMaths  Před měsícem +2

      Thanks! Glad you enjoyed.

    • @akshat9188
      @akshat9188 Před měsícem

      @@OscgrMaths come on man i am preparing for jee and this was not the hardest , again in india these are common ideas

  • @aryanshriv6510
    @aryanshriv6510 Před měsícem

    Bro, I have cleared JEE and am currently studying at one of the top colleges in India, IIT KGP, basically the MIT or Stanford of India. This question is way too easy, obviously. There are some questions that are easy and some that are hard, but if you want to taste the real flavor of JEE, try the 2016 or 2022 papers, especially old JEE papers like those from 1995 to easy 2000s . Those questions were real gems.

  • @Soham.69
    @Soham.69 Před měsícem +10

    Ohh boy this is not even the hardest limit as a sum integration question lol
    Jee main (simpler version of jee advanced) has had harder limit as a sum question than this

    • @OscgrMaths
      @OscgrMaths  Před měsícem +3

      What is the hardest JEE question you've ever seen? I might do that one next!

    • @Soham.69
      @Soham.69 Před měsícem +4

      @@OscgrMaths czcams.com/users/live8erjxlTASgo?feature=shared
      This question is considered to be the toughest one, it uses the same concept of limit of a sum by integration which you talked about in this video

  • @forgottenwishes7775
    @forgottenwishes7775 Před měsícem +11

    Loved it, fun problem and crystal clear explanations. So glad I found your channel!

    • @OscgrMaths
      @OscgrMaths  Před měsícem

      @@forgottenwishes7775 Thanks so much! Really glad you enjoyed.

  • @draido-dev
    @draido-dev Před měsícem +5

    that ain't no exam that's projecteuler 😭

  • @studyingpuppets8916
    @studyingpuppets8916 Před měsícem +2

    TRY OUT THE INTEGRATION PROBLEM FROM JEE ADV 2008 IT WAS A NIGHTMARE FOR MANY AS FAR AS I HAVE HEARD FROM OTHERS :)))

    • @OscgrMaths
      @OscgrMaths  Před měsícem +1

      @@studyingpuppets8916 Okay thank you!!

    • @studyingpuppets8916
      @studyingpuppets8916 Před měsícem +2

      @@OscgrMaths welcome and also one from jee adv 2010 paper 1 on integration.

  • @manijha1
    @manijha1 Před měsícem +1

    By the way I do not get the argument for Sn to be monotonically increasing. Sure, with higher n, we are adding more positive terms, but we are also adding *different* terms, probably smaller terms because we have a n^2 in the denominator

  • @DanishKoul-om4rp
    @DanishKoul-om4rp Před měsícem +1

    I was just solving this pyq and it was discussed in our class today 😂😂😂

  • @Aman-b5k
    @Aman-b5k Před měsícem +1

    ❤thanks buddy for accepting my request❤ you remembered me? ☺️

    • @OscgrMaths
      @OscgrMaths  Před měsícem

      @@Aman-b5k Of course! I'm going to do even more soon hopefully. Thanks for such a good request.

  • @Mandar225
    @Mandar225 Před měsícem +4

    remember solving this in class, it was so much fun!

  • @ionex18
    @ionex18 Před měsícem +25

    This is definitely not the hardest jee adv problem...
    The only tricky part is knowing Sn < S∞ and after that it's a really standard problem which most of the students can solve in 1 min

    • @OscgrMaths
      @OscgrMaths  Před měsícem +2

      @@ionex18 Fair enough - do you have any harder ones I could cover on the channel 😁? I found this one from looking around on stack exchange for hardest maths questions from the JEE but if there's even harder I'd love to see! Thanks for the comment.

    • @Rudrubro11
      @Rudrubro11 Před měsícem +8

      JEE Adv 2019 probability question!

    • @Oproa
      @Oproa Před měsícem +1

      Jee adv famous limit question (you can search on yt u,will easily find it ​@@OscgrMaths

    • @OscgrMaths
      @OscgrMaths  Před měsícem +2

      @@Rudrubro11 Thanks!

    • @ionex18
      @ionex18 Před měsícem +4

      @@OscgrMaths Oh yea sure...
      You can solve jee adv 2016 paper 1 question 43...
      Though it's similar question on Riemann summation... Many people regard it as the toughest question....
      Or you can check out other questions in 2016 coz it was the toughest jee adv paper...

  • @shubhankardatta2437
    @shubhankardatta2437 Před měsícem

    How did you deduce that S_n < S_∞ ?
    From what I see, the Riemann sum of 1/n. f(k/n), k running from 1 to n gives a bigger value than the actual area under the curve y=f(x), because it is the Riemann upper sum. Am I missing anything??
    Edit: I got it. Since f(x) is a decreasing function in the interval [0,1], so the Riemann sum actually gives us the Lower Riemann Sum.

  • @dhirurajput8692
    @dhirurajput8692 Před měsícem +1

    Observe
    n>=k
    =》n^2+nk+k^2>= 3k^2
    =》1/n^2+nk+k^2

  • @mitjain70
    @mitjain70 Před měsícem +2

    🎉easier ,this are training q for those who prepare for jee adv

  • @gregoriousmaths266
    @gregoriousmaths266 Před měsícem +10

    This problem looks familiar...
    I was gonna say that n=1 thing but I got beaten to it lol, nonetheless this is a crazy solution imagine having to see that Riemann sum in the middle of the exam
    Also congrats on 2k

    • @OscgrMaths
      @OscgrMaths  Před měsícem

      @@gregoriousmaths266 Thank you! Yeah Riemann sum part is totally crazy.

    • @krishpandey854
      @krishpandey854 Před měsícem +2

      @@OscgrMaths totally depends on your exposure. As an Indian student, we solve lots of problems pertaining to reinmann sums. So, literally the first thing, I would observe when i see this question is, hey that is the reimann sum without the limit, and the options clearly indicate that some integration is probably involved.
      To set up the inequality, I drew a rough graph of 1/(1+x+x^2). The total area under the curve is the value of Sn as n approaches inf i.e. rectangular strips are infinitely small.
      Sn with n being finite represents an approximate area under the curve. When you start k from 1 to n, the summation represents the area with rectangles of base length 1/n. It can be graphically observed that this area is less then the actual area under the curve, similar logic for second part

  • @adampickarski5172
    @adampickarski5172 Před měsícem +9

    I am not sure how you readily conclude S_n is an increasing sequence of numbers. if you take the difference of S_{n+1} and S_n, you do get an additional term at the top (at n+1) but the difference of the other terms must be dealt with

    • @OscgrMaths
      @OscgrMaths  Před měsícem +1

      Yeah there was a bit more work behind the scenes here.. sorry for brushing over it!

  • @RimuruTempest-jr7de
    @RimuruTempest-jr7de Před měsícem

    Great content; do not clickbait though.
    Jee is nowhere the toughest exam, and this is definitely no the toughest question on jee.

  • @arjishchowdhury1898
    @arjishchowdhury1898 Před měsícem

    If u want the hardest problem, check the jee adv 2016 limits problem and jee adv 2020 matrices problem.

    • @OscgrMaths
      @OscgrMaths  Před měsícem +1

      @@arjishchowdhury1898 Okay great thanks!

  • @alexandersanchez9138
    @alexandersanchez9138 Před měsícem +5

    If the inequality is supposed to hold for all values of n, just substitute n=1. Then, since pi/3sqrt(3) ~ 22/21sqrt(3) > 1/sqrt(3) > 1/3 = S1, the answer is A.

    • @OscgrMaths
      @OscgrMaths  Před měsícem

      @@alexandersanchez9138 This is a nice strategy.

    • @alexandersanchez9138
      @alexandersanchez9138 Před měsícem +3

      @@OscgrMaths Yeah. It's certainly not a legit solve, but it cheeses the multiple-choice format nicely. (I understand these poor students are ALSO severely time-crunched on this exam.)

    • @OscgrMaths
      @OscgrMaths  Před měsícem +1

      @@alexandersanchez9138 Definitely a huge time pressure!!!

    • @FRANKONATOR123
      @FRANKONATOR123 Před měsícem +1

      Yeah when I first saw the problem I thought of exactly that. Either way, enjoyed the video 😂

  • @DemonBoy-ks4nt
    @DemonBoy-ks4nt Před měsícem

    bro isnt that Sn>S∞ because the whole sum is going to zero as n=∞
    and if i put the values of n as 1 and 2
    i got S1>S2 = 3/4>9/7

  • @samarthbagwe1736
    @samarthbagwe1736 Před měsícem

    This is not the hardest question
    Nobody sees this any hard
    The real hard questions are completely different

  • @Yashodhan1917
    @Yashodhan1917 Před měsícem +4

    I put n=2, and summed two terms to check. That's how you should do it in the exam.

    • @shivangsingh5834
      @shivangsingh5834 Před měsícem +1

      It's not about how you do it in exam. It's about how your intuition works your perspective of looking the problem look how beautifully I explained.

    • @Yashodhan1917
      @Yashodhan1917 Před měsícem

      @@shivangsingh5834 Thank you professor. I know what math is about. Have you cracked JEE by any chance?

    • @user-en5vj6vr2u
      @user-en5vj6vr2u Před měsícem

      Step your game up buddy. It said n may be equal to 1, so set it equal to 1!

    • @Yashodhan1917
      @Yashodhan1917 Před měsícem

      @@user-en5vj6vr2u Haha yes. I did not miss that, but there is always a small possibility that the formula doesn't work for border cases. Generally in such cases the question puts a limit on the value of two; but since you are supposed to solve this analytically they might intentionally skip it. So it is safer to put n=2, but in most cases n=1 may also work; and in this problem it should work (I guess).

  • @dannyfilmss
    @dannyfilmss Před měsícem +2

    I cant stop watching your videos! 😂 so interesting, when I grow up i want to be a mathematician

    • @dannyfilmss
      @dannyfilmss Před měsícem +1

      By the way students take this on a computer in the actual exam

    • @OscgrMaths
      @OscgrMaths  Před měsícem +1

      Thats great!! So glad you're enjoying.

  • @nihalpurohit1499
    @nihalpurohit1499 Před měsícem +1

    n=1 simple

  • @symmetricfivefold
    @symmetricfivefold Před měsícem +1

    had trouble understanding riemann sums
    walked out understood everything

  • @bananablitzcoding1696
    @bananablitzcoding1696 Před měsícem +4

    I used the integral test right from the get go and was able to solve it with a tighter bound I think. However, the integral was a bit of a pain and led to a less neat solution.

  • @O_of_N
    @O_of_N Před měsícem +2

    great work man i loved the vid and the explanation was great

    • @OscgrMaths
      @OscgrMaths  Před měsícem +1

      @@O_of_N Thanks so much! Glad you enjoyed.

  • @JOSHUVASRINATH
    @JOSHUVASRINATH Před měsícem +6

    I compared it with base problem with inequality

    • @OscgrMaths
      @OscgrMaths  Před měsícem

      @@JOSHUVASRINATH Nice!

    • @samrubenabraham6979
      @samrubenabraham6979 Před měsícem

      @@OscgrMaths FYI, JEE is "Joint Entrance Examination" and the question must be probably from JEE Advanced, the one that is used to determine the entry of the participants into IIT's and IISER's. JEE Main is a slightly easier exam and is for entry into NITs, IIIT's and some other such institutes (i.e., those excluding IITs and such institutes) and also serve as an elimination round for the participants to attend JEE Advanced. Still, thanks for covering the question.

    • @PXO005
      @PXO005 Před měsícem

      @@samrubenabraham6979slight correction: IISERS closed the JEE channel recently making the only way of entry to any IISER through their own entrance exam, but iisc still accepts jee scores.

  • @udaykaranwal1662
    @udaykaranwal1662 Před měsícem +1

    I remember solving this Q and more like this one back in 11th grade, we are actually taught riemann sum as a part of problem solving techniques for questions involving series and sequences, this is actually among the easier ones

    • @vaanivijay6552
      @vaanivijay6552 Před měsícem

      Damn bro u in IIT now???

    • @udaykaranwal1662
      @udaykaranwal1662 Před měsícem +1

      @@vaanivijay6552 fortunately, yep

    • @vaanivijay6552
      @vaanivijay6552 Před měsícem

      @@udaykaranwal1662 wow congratulations! Which year did u write IIT JEE? Which iit r u in now? Any tips for us aspirants bro?

    • @udaykaranwal1662
      @udaykaranwal1662 Před měsícem

      @@vaanivijay65522022, IIT Delhi, do PYQs, give paper during the same time as the exams, focus on a strong theoretical foundation and doing conceptually tough questions, adv pyqs are a must then you can move on to other books

  • @MelkorNoir
    @MelkorNoir Před měsícem

    Congratulations on another fantastic video! Your style is lucid and engaging, and you've clearly thought deeply about the exact order to present each step. This is the third video I've seen of yours - I've liked and subscribed :)

    • @OscgrMaths
      @OscgrMaths  Před měsícem +1

      Thanks so much! That really means a lot.

  • @arkoprabhamisra8417
    @arkoprabhamisra8417 Před měsícem +1

    Great video

  • @dolichakraborty6020
    @dolichakraborty6020 Před měsícem

    It is not the hardest of india also not the hardest problem😂😂😂😂

    • @Midnight_9595
      @Midnight_9595 Před měsícem

      Then what is the toughest competitive exam of India?

  • @Begula34
    @Begula34 Před měsícem +2

    Keep in mind that u have to solve this in 5 min only

    • @OscgrMaths
      @OscgrMaths  Před měsícem +1

      I know, that's why the JEE is so tough!

  • @sohampine7304
    @sohampine7304 Před měsícem +1

    Brother please do more JEE questions we Indians love you😊😊

    • @OscgrMaths
      @OscgrMaths  Před měsícem

      @@sohampine7304 Will do!! Thanks for the comment.

  • @bingchilling4717
    @bingchilling4717 Před měsícem +3

    didnt watch the video yet cuz i wanted to do it myself but doesnt the sum become a riemann integral and it would be integral from 0 to1 of 1/(1+x+x^2) which is kinda easy to solve

    • @OscgrMaths
      @OscgrMaths  Před měsícem

      Yes!! That's the satisfying part, nice spot.

    • @bingchilling4717
      @bingchilling4717 Před měsícem +1

      @@OscgrMaths thanks you love this sums please cover more problems from jee i never heard of it

    • @OscgrMaths
      @OscgrMaths  Před měsícem

      @@bingchilling4717 Will do!

  • @RuchiReddy-ob7ju
    @RuchiReddy-ob7ju Před měsícem

    Which university are you from bro

  • @red0guy
    @red0guy Před měsícem +1

    Your explanation of why S_n is increasing is shakey. Yes you keep adding up more and more parts, but those parts become smaller and smaller. You could compute S_n+1 - S_n and match the k term from S_n+1 with the k+1 term from S_n and see that indeed the difference of those two terms is positive.

    • @MelkorNoir
      @MelkorNoir Před měsícem

      ​@@red0guy I see my mistake now. Each term in the sum depends on n, but so does the upper limit of the sum. I agree with you, this is a little trickier than it first appears.

  • @unbelievable961
    @unbelievable961 Před měsícem +2

    Can you please tell me in which year this question came i jee?? And I wanna make sure that are you taking about the jee advance for high school students?!

    • @OscgrMaths
      @OscgrMaths  Před měsícem +1

      This one is from 2008! Hope that helps.

  • @shivx3295
    @shivx3295 Před měsícem

    This is preety easy and also i think this is not the toughest jee question

  • @niom-nx7kb
    @niom-nx7kb Před měsícem

    Would be nice if more hard questions like these are made as your videos :)

  • @Tosi31415
    @Tosi31415 Před měsícem +2

    well well well..... if it has to hold for any integer n, it also has to work for n=1, i plug that and I get 1/3,which is less than that number, so i solved it lol

    • @Tosi31415
      @Tosi31415 Před měsícem +1

      not proved obviously but since it has to be either a or b, it must be a by this reasoning

    • @OscgrMaths
      @OscgrMaths  Před měsícem

      Nice! Very smart...

    • @Tosi31415
      @Tosi31415 Před měsícem +2

      @@OscgrMaths lazyness can bring you far sometimes

  • @brickie9816
    @brickie9816 Před měsícem

    why does S_n < S_inf ? as n approaches infinity number of terms increases but they are also smaller

    • @OscgrMaths
      @OscgrMaths  Před měsícem +1

      But even if they are smaller it will still make a slight increase each time, meaning the maximum value must be as n goes to infinity since there are no more terms left to add.

    • @brickie9816
      @brickie9816 Před měsícem

      @@OscgrMaths ah I see, thank you!

    • @OscgrMaths
      @OscgrMaths  Před měsícem +1

      It's my bad for skipping over that in the video, there is a way to be more rigorous about it. Thanks for raising the question!

    • @LinaWainwright
      @LinaWainwright Před měsícem

      ​​@@OscgrMaths This reasoning is incorrect. Here's a simple counterexample: The exact same sum but switch the bounds from 1->n to 0->n-1.
      Here you have to identify the summation as a right Riemann Sum, and because the function f is continuous and decreasing on the interval [0; 1], then the sequence is approaching the integral from below, thus S_nn-1, you can see that the summation is a left Riemann Sum, and since f is continuous and decreasing on [0; 1], then the sequence is approaching the integral from above, that is, S_inf

    • @OscgrMaths
      @OscgrMaths  Před měsícem +1

      @@LinaWainwright That's what the challenge question at the end is all about - it's the exact same sum but starting at a different place. Sorry for the explanation in the video and thanks for typing this all out!

  • @musicposts1958
    @musicposts1958 Před měsícem

    The question maybe a lil tricky but as questions on same approach have been asked before so it becomes easy to know the solution

  • @ahmedlutfi4894
    @ahmedlutfi4894 Před měsícem +1

    making jee more fun

  • @blisterff6769
    @blisterff6769 Před měsícem

    Can i get why cant i just get n=1 to be a way to solve, since S1=1/3 and we now that √3 1 then
    S1 < π/3√3 and since its just an A, B question then then we can deduce that for all n and say that A is the correct answer

  • @Chonk707
    @Chonk707 Před měsícem

    🙂 thanks but how can we solve this under 3mins

    • @PIYUSH-694
      @PIYUSH-694 Před měsícem

      He is explaoning no one solve in 3 min

    • @OscgrMaths
      @OscgrMaths  Před měsícem

      Shortcut is substituting n=1, given that the inequality has to work for all n.

  • @aadeshr4307
    @aadeshr4307 Před měsícem +1

    I have solve this question while practicing some previous year and I did not even thought it was hard😅 I tried another way it states that the the inequality of SN so we can have n equal to any number so I put n equal to one and I got one third which is smaller but I could be wrong

    • @OscgrMaths
      @OscgrMaths  Před měsícem

      @@aadeshr4307 That's a nice shortcut!

  • @vedantjaiswal6734
    @vedantjaiswal6734 Před měsícem

    As an 11 th greader i tried to understand this but i am not able to hope in couple of month i will 😅
    Btw i am preparing for iit jee

  • @user-hr4vr6oj3v
    @user-hr4vr6oj3v Před měsícem +7

    Nice explanation bro - never seen anything like this but still managed to follow pretty well. Keep it up boss

    • @OscgrMaths
      @OscgrMaths  Před měsícem

      @@user-hr4vr6oj3v Thank you! Really glad you enjoyed, thanks for the comment.

  • @cliche_ayush
    @cliche_ayush Před měsícem

    Cant it be dn using telescoping?

    • @OscgrMaths
      @OscgrMaths  Před měsícem

      @@cliche_ayush I'm not quite sure what you mean but it's definitely worth a shot - if you solve it with a new method let me know and I'd love to see!

  • @Er4serOP
    @Er4serOP Před měsícem +1

    I assume you could just re index the challenge question and tackle it a similar way to the original?

    • @OscgrMaths
      @OscgrMaths  Před měsícem

      @@Er4serOP Yeah exactly... that's a good place to start!

  • @jcfgykjtdk
    @jcfgykjtdk Před měsícem +1

    Can this be solved without knowing riemann's sum?

    • @OscgrMaths
      @OscgrMaths  Před měsícem

      Riemann sums is how I did it but if you manage to find another approach I'd love to know!!

  • @marcsameh7392
    @marcsameh7392 Před měsícem +1

    Bro is a genius

    • @OscgrMaths
      @OscgrMaths  Před měsícem +1

      I wouldn't say that!! But thanks for the comment.

  • @basiliskprime9952
    @basiliskprime9952 Před měsícem +1

    This is so simple.
    sum(n, k=1) n/(n^2+kn+k^2)
    => sum(n, k=1) n/((n^3-k^3)(n-k))
    => sum(n, k=1) n(n-k)/(n^3-k^3)
    now that it is simplified...
    when k reaches n, n-k = 0 so we can ignore the last term(except when taken as 1*0/0 instead take as lim x->0 1*0/x = 0/x = 0
    n=1
    => sum(1, k=1) 1(1-1)/(1^3-1^3)
    use lim x->0 1*0/x = 0/x = 0
    0 2(2-1)/(2^3-1^3) + 2(2-2)/(2^3-2^3)
    the second term or the nth term is why we use lim x->0 1*0/x = 0/x = 0
    => 2/73 is enough

  • @two697
    @two697 Před měsícem

    Not to be that guy but i don't this was too hard. Riemann sums seem the obvious way to go. Also, from the problem set up, you can guess that the integral is equal to pi/sqrt(27) and skip half the work

    • @julianbruns7459
      @julianbruns7459 Před měsícem +1

      It becomes hard when you consider the limited time and the education level of those who take it.

  • @MathematicFanatic
    @MathematicFanatic Před měsícem +1

    Does the statement S_n < number mean that the limit of S_n is less than the number?

    • @OscgrMaths
      @OscgrMaths  Před měsícem

      Hey what do you mean with limit of S_n? Thanks for the question!

    • @MathematicFanatic
      @MathematicFanatic Před měsícem +1

      @@OscgrMaths I don't know what S_n < pi/3sqrt3 means because S_n is not a number but a sequence. Does it mean instead the limit as n goes to infinity of S_n < pi / 3sqrt3, and does it always mean that when you have a sequence in an inequality or equation?

    • @angelvalenciapadilla627
      @angelvalenciapadilla627 Před měsícem +1

      @@MathematicFanaticApparently in this problem Sn is a sum not a sequence, I also got confused by the notation

    • @OscgrMaths
      @OscgrMaths  Před měsícem

      @@MathematicFanatic Whatever you choose n as is the upper limit of your sum, so you're looking for an inequality that holds for all n in the natural numbers. Hope this helps!

    • @MathematicFanatic
      @MathematicFanatic Před měsícem

      @@OscgrMaths By this interpretation, the choices are (S_n < pi/3sqrt3 for all values of n) and (S_n > pi/3sqrt3 for all values of n). This makes the problem extremely easy because I can just check that S_1 = 1/3 so I know that the second option is false so it must be the first option.

  • @rudraboghani9006
    @rudraboghani9006 Před měsícem

    How old are you?

  • @matemindak384
    @matemindak384 Před měsícem +2

    This was sick, I've never seen a problem like this before

    • @OscgrMaths
      @OscgrMaths  Před měsícem +1

      Thanks! I thought it was quite unique too, which is why I wanted to share.

  • @PooshanHalder
    @PooshanHalder Před měsícem +2

    this is not the hardest problem in jee but nonethanless this is an interesting question(and one of the challenging ones to solve in those restricting time bound situations)

    • @OscgrMaths
      @OscgrMaths  Před měsícem

      @@PooshanHalder Yeah definitely! I found it by looking up hardest JEE question but I'm sure there will be others that are even harder!!

    • @milianxhighlights
      @milianxhighlights Před měsícem

      ​@@OscgrMaths There is a question from JEE ADV 2016 paper You'll love to solve that

  • @DrMeikoHayakawa
    @DrMeikoHayakawa Před měsícem +1

    It might difficult for Indians, but not the most difficult one throughout the world. The difficulty level is clearly mediocre.

    • @SahilRawal-fe7qc
      @SahilRawal-fe7qc Před měsícem

      What do you even mean by that😂😂 the guy who have solved this sum called it harder which means It's hard for him Indians can solve this question within minutes ever heard about JEE advanced exam?😂😂 Sundar ceo of Google gave that exam crack it and got into iit(dream college of many jee aspirants)and now you know he is ceo of Google it's second hardest exam of the entire world just after gaokao from china the difficulty of that paper is so much that scoring 40 percent in the exam will get you in iit 😂😂 I bet you can't even score 1percent in that exam you will be in negative😂😅

  • @anas.aldadi
    @anas.aldadi Před měsícem +1

    Skill issue

    • @SalmonForYourLuck
      @SalmonForYourLuck Před měsícem +3

      How old are you

    • @anas.aldadi
      @anas.aldadi Před měsícem

      @@SalmonForYourLuck mentally or chronologically?

    • @joaquingomez9226
      @joaquingomez9226 Před měsícem +3

      @@anas.aldadi This answer makes it clear, thanks!

    • @anas.aldadi
      @anas.aldadi Před měsícem

      @@joaquingomez9226 ​ @SalmonForYourLuck I am joking chill guys, I literally subbed to his channel since months and i saw all his vids i like his content, this is my first comment and i thought it would be funny to write it 😮‍💨but i guess in internet people take everything seriously

    • @anas.aldadi
      @anas.aldadi Před měsícem

      ​@@joaquingomez9226 ​ @SalmonForYourLuck I am joking chill guys, I literally subbed to his channel since months and i saw all his vids and i like his content. This is my first comment and i thought it would be funny to write it 😮‍💨but i guess in internet people take everything seriously